MODULE 4 Flashcards

1
Q

Lady, a 23-year-old nurse, complained of fever and maculopapular rashes for 4 days. Associated symptoms include cough, coryza, and conjunctivitis. Since the start of the pandemic, she has been facing COVID-19 patients almost every day. Hospital work has been hectic and healthcare workers are overworked and underpaid. As such, Lady hasn’t been eating healthy, and she hasn’t had decent sleep for the past few months. She has no history of measles, German measles, chickenpox, mumps, and other diseases as a child. She’s currently not pregnant. Which of the following is the most likely diagnosis of Lady’s condition?

A. Scarlet fever
B. German measles
C. COVID-19
D. Measles

A

D. Measles

How well did you know this?
1
Not at all
2
3
4
5
Perfectly
2
Q

Accumulation of pleural fluid occurs in the following settings

A. Increased osmotic pressure, as in nephrotic syndrome
B. Decreased hydrostatic pressure, as in congestive heart failure
C. Increased intrapleural negative pressure, as in atelectasis
D. Decreased vascular permeability, as in pneumonia

A

C. Increased intrapleural negative pressure, as in atelectasis

How well did you know this?
1
Not at all
2
3
4
5
Perfectly
3
Q

What should be done once one or more of the exudative criteria are met and the patient is clinically thought to have a condition producing transudative effusion?

A. The difference between the protein levels in the serum and the pleural fluid should be measured
B. Treat the patient as one with a condition producing exudative effusion
C. Thoracentesis must be performed for cytology
D. Lung biopsy is recommended to rule out metastatic spread

A

A. The difference between the protein levels in the serum and the pleural fluid should be measured

How well did you know this?
1
Not at all
2
3
4
5
Perfectly
4
Q

Which of the following is NOT caused by coronaviruses?

A. COVID-19
B. Respiratory syncytial virus infection
C. Middle East respiratory syndrome
D. Severe acute respiratory syndrome

A

B. Respiratory syncytial virus infection

How well did you know this?
1
Not at all
2
3
4
5
Perfectly
5
Q

A middle male aged alcoholic with fever, cough, sputum production, and chest pain. Upon physical examination, he had poor dentition, and cavernous breath sounds on lung auscultation. A diagnosis of lung abscess is entertained. Which of the following is the best treatment for this case?

A. Clindamycin
B. Metronidazole
C. Cloxacillin
D. Ceftriaxone

A

A. Clindamycin

How well did you know this?
1
Not at all
2
3
4
5
Perfectly
6
Q

Orchitis and epididymitis is seen in young adult males as a complication of certain RNA viral infection. This disease usually presents with

A. generalized rash
B. nonsuppurative enlargement of parotid glands
C. fever
D. Koplik spots

A

B. nonsuppurative enlargement of parotid glands

How well did you know this?
1
Not at all
2
3
4
5
Perfectly
7
Q

A middle male aged alcoholic presented with fever, cough, sputum production, and chest pain. Upon physical examination, he had poor dentition, and cavernous breath sounds on lung auscultation. Which part of the history and PE point to a primary lung abscess?

A. Middle male aged alcoholic
B. Cavernous breath sounds
C. Fever, cough, sputum production
D. Poor dentition

A

A. Middle male aged alcoholic

How well did you know this?
1
Not at all
2
3
4
5
Perfectly
8
Q

The current minimal acceptable duration of treatment for all adults and children with active tuberculosis is

A. 9 months
B. 12 months
C. 3 months
D. 6 months

A

D. 6 months

How well did you know this?
1
Not at all
2
3
4
5
Perfectly
9
Q

What is Legionnaires’ disease characterized by?

A. Headache, muscle aches, high fever, confusion, shaking chills, dry cough that later becomes productive
B. Sore throat, difficulty swallowing, fever, whitish patches on the red throat, swollen lymph nodes
C. Mild upper respiratory symptoms, followed by bursts of violent dry coughing
D. Symptoms vary greatly and are unpredictable

A

A. Headache, muscle aches, high fever, confusion, shaking chills, dry cough that later becomes productive

How well did you know this?
1
Not at all
2
3
4
5
Perfectly
10
Q

The following are risk factors in acquiring Multidrug Resistant Gram-Negative Bacteria and MRSA, except:

A. erythematous rash
B. bedridden
C. use of antibiotics in previous 90 days
D. tube feeding

A

A. erythematous rash

How well did you know this?
1
Not at all
2
3
4
5
Perfectly
11
Q

Which of the following statements about pathogenesis of Legionella pneumophila is true?

A. The causative organisms promote their own uptake by macrophages
B. The causative agent is quickly destroyed by the alveolar macrophages
C. The organism establishes a latent infection in most people
D. The causative organisms avoid the immune system by producing a capsule

A

A. The causative organisms promote their own uptake by macrophages

How well did you know this?
1
Not at all
2
3
4
5
Perfectly
12
Q

Why can antigenic shifts cause pandemics?

A. With antigenic shift, repeated mutations cause a gradual change in the HA and/or NA spikes, so that antibodies against the virus become much less effective
B. Antigenic shifts can lead to extreme changes in the HA and/or NA spikes, leading to pandemics because those viruses can enter different types of body cells
C. Antigenic shifts can lead to the generation of novel flu viruses by genetic reassortment, leading to pandemics because no one is immune to the new virus
D. Antigenic shifts actually cause minor changes in flu vaccines and do not cause pandemics, antigenic drifts lead to global pandemics

A

C. Antigenic shifts can lead to the generation of novel flu viruses by genetic reassortment, leading to pandemics because no one is immune to the new virus

How well did you know this?
1
Not at all
2
3
4
5
Perfectly
13
Q

Antibiotics like penicillin can act on bacterial _______ are not effective for the treatment of Mycoplasma pneumoniae.

A. cell membrane synthesis
B. cell wall synthesis
C. protein synthesis
D. DNA synthesis

A

B. cell wall synthesis

How well did you know this?
1
Not at all
2
3
4
5
Perfectly
14
Q

The following Mycobacterium species cause tuberculosis in humans, EXCEPT:

A. Mycobacterium bovis
B. Mycobacterium tuberculosis
C. Mycobacterium microti
D. Mycobacterium africanum

A

C. Mycobacterium microti

How well did you know this?
1
Not at all
2
3
4
5
Perfectly
15
Q

Mycoplasmal pneumonia is a _______, with an incubation period of _______.

A. serious disease requiring hospitalization; 2-3 weeks
B. serious disease requiring hospitalization; 2-3 days
C. generally mild form of pneumonia; 2-3 weeks
D. generally mild form of pneumonia; 2-3 days

A

C. generally mild form of pneumonia; 2-3 weeks

How well did you know this?
1
Not at all
2
3
4
5
Perfectly
16
Q

The cardinal histologic change in all abscesses is

A. hemorrhagic changes in the alveoli
B. suppurative destruction of the lung parenchyma within the central area of cavitation
C. pulmonary edema evident throughout the lung parenchyma
D. pleural effusion that is transudative in nature

A

B. suppurative destruction of the lung parenchyma within the central area of cavitation

How well did you know this?
1
Not at all
2
3
4
5
Perfectly
17
Q

Which of the following is true regarding treatment of URIs?

A. Antibiotics have no role in the treatment of uncomplicated nonspecific URI
B. Antibiotics have a role in the treatment of uncomplicated nonspecific URI
C. Antibiotics have no role in the treatment of URI
D. None of the options is true

A

A. Antibiotics have no role in the treatment of uncomplicated nonspecific URI

How well did you know this?
1
Not at all
2
3
4
5
Perfectly
18
Q

A patient had a subacute illness of fever, cough, and chest pain associated with weight loss, a brisk leukocytosis, and mild anemia. Presence of free pleural fluid was demonstrated with a lateral decubitus radiograph. Bacterial pneumonia was considered. What kind of pleural effusion does the patient have?

A. Effusion due to heart failure
B. Effusion secondary to malignancy
C. Effusion secondary to pulmonary embolization
D. Parapneumonic effusion

A

D. Parapneumonic effusion

How well did you know this?
1
Not at all
2
3
4
5
Perfectly
19
Q

Which of the following situations regarding the vaccine Bacillus Calmette Guerin is correct?

A. Does not interfere with the interpretation of the tuberculin skin test
B. Most effective in preventing severe TB infection in children
C. Induces a long lasting immunity against tuberculosis
D. Provides consistent protection against contracting pulmonary tuberculosis in adulthood

A

B. Most effective in preventing severe TB infection in children

How well did you know this?
1
Not at all
2
3
4
5
Perfectly
20
Q

No practical preventive measures exist for mycoplasmal pneumonia, except for:

A. treatment with cephalosporins or related drugs
B. childhood vaccination
C. avoiding overcrowding in schools and military facilities
D. prophylactic use of antibiotics

A

C. avoiding overcrowding in schools and military facilities

How well did you know this?
1
Not at all
2
3
4
5
Perfectly
21
Q

What is a typical infectious dose of Mycoplasma pneumoniae?

A. At least a million inhaled cells
B. Only a few inhaled cells
C. About a thousand inhaled cells
D. No cells need to be inhaled to cause infection

A

B. Only a few inhaled cells

How well did you know this?
1
Not at all
2
3
4
5
Perfectly
22
Q

A negative tuberculin skin test can be seen in which of the following circumstances?

A. Anergy
B. Early acute infection of tuberculosis
C. Advanced and overwhelming tuberculosis
D. All of the options will give a negative tuberculin test

A

D. All of the options will give a negative tuberculin test

How well did you know this?
1
Not at all
2
3
4
5
Perfectly
23
Q

A 65-year-old came to your clinic due to chronic cough, body malaise, and easy fatigability. As a young man, a tuberculin skin test was done on him when he was assigned in a foreign country and showed a positive reaction. A sputum examination and chest x-ray as subsequently requested. He came back a week later with the following results: Sputum (done for 3 consecutive days). No acid-fast tubercle bacilli found.
CXR: coarse reticulonodular densities involving the posterior segments of the right upper lobe and the superior segment of the left lower lobe. An endobronchial “tree-in-bud” appearance is also seen. Based on the above findings, the patient is placed under which category of TB Classification System?

A. Class 3 – clinically active TB
B. Class 2 – TB infection but no disease
C. Class 4 – previous TB disease not clinically active
D. Class 1 – positive exposure but no evidence of TB infection

A

A. Class 3 – clinically active TB

How well did you know this?
1
Not at all
2
3
4
5
Perfectly
24
Q

The following are true regarding treatment of URIs, except

A. Decongestants and nonsteroidal anti-inflammatory drugs (NSAIDs) can be given for symptom management
B. In healthy volunteers, a single course of a commonly prescribed antibiotic like azithromycin can result in macrolide resistance in oral streptococci many months later
C. Clinical trials of zinc, vitamin C, other alternative remedies have revealed consistent benefits in the treatment of nonspecific URI
D. In the absence of clinical evidence of bacterial infection, treatment remains entirely symptom-based

A

C. Clinical trials of zinc, vitamin C, other alternative remedies have revealed consistent benefits in the treatment of nonspecific URI

How well did you know this?
1
Not at all
2
3
4
5
Perfectly
25
Q

Which of the following is the most common cause of URI?

A. Rhinovirus
B. Streptococcus pneumoniae
C. Adenovirus
D. Coronavirus

A

A. Rhinovirus

How well did you know this?
1
Not at all
2
3
4
5
Perfectly
26
Q

Effective preventive methods for avoiding the common cold include all of the following, EXCEPT

A. avoiding crowds
B. prophylactic antibiotics
C. not touching one’s face
D. hand washing

A

B. prophylactic antibiotics

How well did you know this?
1
Not at all
2
3
4
5
Perfectly
27
Q

Mycoplasma pneumoniae is spread by

A. aerosol droplets
B. direct contact
C. fomite contamination
D. contaminated water

A

A. aerosol droplets

How well did you know this?
1
Not at all
2
3
4
5
Perfectly
28
Q

Projecting from the outer envelope of the influenza virus are two glycoproteins called

A. neuraminidase and hyaluronidase
B. hyaluronidase and coagulase
C. hemagglutinin and neuraminidase
D. leukocidin and hemolysin

A

C. hemagglutinin and neuraminidase

How well did you know this?
1
Not at all
2
3
4
5
Perfectly
29
Q

The following are true of lung abscesses, except for:

A. Lung abscesses represents necrosis and cavitation of the lung following microbial infection
B. Lung abscesses are usually characterized as either primary (~80% of cases) or secondary
C. Secondary lung abscesses usually arise from aspiration, are often caused principally by anaerobic bacteria, and occur in the absence of an underlying pulmonary or systemic condition
D. Lung abscesses can be single or multiple but usually are marked by a single dormant cavity more than 2 cm in diameter

A

C. Secondary lung abscesses usually arise from aspiration, are often caused principally by anaerobic bacteria, and occur in the absence of an underlying pulmonary or systemic condition

How well did you know this?
1
Not at all
2
3
4
5
Perfectly
30
Q

Jose, a 75-year-old retired politician, is a 50-pack-year smoker and had been diagnosed with COPD since he was 60 years old. He takes his maintenance medications with good compliance. Two days PTA, Jose experienced fever, cough, shortness of breath with increased sputum production. His maintenance medications provided no relief at this time. He was seen by you in the OPD for consult of the above symptoms. Your assessment currently was CAP-MR, COPD in acute exacerbation. Which of the following etiologic agents could have caused Jose’s COPD exacerbation?

A. Streptococcus pneumoniae
B. Haemophilus influenzae
C. Staphylococcus pneumoniae
D. Moraxella catarrhalis

A

D. Moraxella catarrhalis

How well did you know this?
1
Not at all
2
3
4
5
Perfectly
31
Q

The anti-tuberculosis drug, Isoniazid, is directed to which population of tubercle bacilli?

A. Those that are dormant
B. Those actively multiplying extracellular bacilli
C. Those characterized by spurts of growth-interspersed with periods of dormancy
D. Those that are growing more slowly

A

A. Those that are dormant

How well did you know this?
1
Not at all
2
3
4
5
Perfectly
32
Q

Which of the following is true about pneumonia?

A. Staphylococcus aureus is the most common cause of community acquired pneumonia
B. Streptococcus pneumoniae is associated with a high incidence of lung abscess and empyema
C. Intravenous drug users are at high risk for development of streptococcal pneumonia in association with endocarditis
D. Streptococcus pneumoniae is not part of the endogenous flora in 80% of adults, and therefore false positive results may be obtained

A

D. Streptococcus pneumoniae is not part of the endogenous flora in 80% of adults, and therefore false positive results may be obtained

How well did you know this?
1
Not at all
2
3
4
5
Perfectly
33
Q

The best way to speed up recovery from a common cold is

A. to dose the patient with paracetamol to keep the fever down. Reducing fever speeds up recovery.
B. take an antibiotic such as penicillin that will rid the body of the cold virus.
C. to take 1,000 mg of vitamin C every day during the illness. Vitamin C destroys cold viruses.
D. let the immune system do its job. Several of the treatments listed may actually prolong the recovery time.

A

no answer in coffee

How well did you know this?
1
Not at all
2
3
4
5
Perfectly
34
Q

Which of the following are typical signs and symptoms of mycoplasmal pneumonia when symptomatic?

A. Sore throat, difficulty swallowing, fever, whitish patches on the red throat, swollen lymph nodes
B. Pseudomembrane in throat, sore throat, fever, fatigue
C. Sore throat, chills, fever, headache, muscle pain, fatigue and dry cough
D. Sudden chills and fever, severe chest pain, pinkish sputum, and sometimes cyanosis

A

C. Sore throat, chills, fever, headache, muscle pain, fatigue and dry cough

How well did you know this?
1
Not at all
2
3
4
5
Perfectly
35
Q

Why is the etiologic agent of Legionnaires’ disease typically detected using immunofluorescence?

A. It is Gram non-reactive
B. It stains poorly with conventional dyes
C. It lacks a peptidoglycan cell wall
D. It produces spores that are fluorescent

A

B. It stains poorly with conventional dyes

How well did you know this?
1
Not at all
2
3
4
5
Perfectly
36
Q

Which of the following statements regarding the global impact of tuberculosis is TRUE?

A. About 1/3 of the world’s population is infected
B. Responsible for the death of more people than any other infectious disease in history
C. Occurs in almost every country in the world
D. All of the options are true

A

D. All of the options are true

How well did you know this?
1
Not at all
2
3
4
5
Perfectly
37
Q

Tuberculosis can develop through which of the following circumstances?

A. Exogenous reinfection
B. Reactivation of latent infection
C. Progression of recently acquired infection
D. All of the options are correct

A

D. All of the options are correct

How well did you know this?
1
Not at all
2
3
4
5
Perfectly
38
Q

Haemophilus influenzae is a pleomorphic, Gram negative organism that occurs in encapsulated and nonencapsulated forms. The following are true about H. influenzae, except:

A. They are less virulent, spread along the surface of the upper respiratory tract, and produce otitis media, sinusitis, and bronchopneumonia
B. H. influenzae pneumonia, which may follow a viral respiratory infection, has a low mortality rate
C. Neonates and children with comorbidities, such as prematurity, malignancy, and immunodeficiency are at high risk for development of invasive infection
D. There are six serotypes of the encapsulated form (types a to f), of which d is the most virulent

A

B. H. influenzae pneumonia, which may follow a viral respiratory infection, has a low mortality rate

How well did you know this?
1
Not at all
2
3
4
5
Perfectly
39
Q

Humans become infected with Mycobacterium tuberculosis most frequently by which route?

A. Contact
B. Inoculation
C. Inhalation
D. Ingestion

A

C. Inhalation

How well did you know this?
1
Not at all
2
3
4
5
Perfectly
40
Q

Influenza is caused by

A. paramyxovirus
B. coronavirus
C. orthomyxovirus
D. cytomegalovirus

A

C. orthomyxovirus

How well did you know this?
1
Not at all
2
3
4
5
Perfectly
41
Q

How is Legionella pneumophila acquired?

A. By bites from infected insect vectors
B. By touching contaminated fomites and surfaces
C. By direct contact with an infected person
D. By inhaling water droplets contaminated with the organism

A

D. By inhaling water droplets contaminated with the organism

How well did you know this?
1
Not at all
2
3
4
5
Perfectly
42
Q

Which is more likely to happen – antigenic DRIFT, or antigenic SHIFT – and why?

A. Antigenic drift and antigenic shift occur at the same frequency. Genetic change is just as likely to occur if one viral strain or more than one strain is/are present
B. Antigenic SHIFT since infection with only a single virus strain is required, and the random mutations happen as the virus replicates in the infected person’s cells
C. Antigenic DRIFT since infection with only a single virus strain is required, and the random mutations happen as the virus replicates in the infected person’s cells
D. Antigenic SHIFT since multiple viruses in a cell at once means more RNA polymerase to copy the RNA, and therefore more possibilities for mistakes to be made (leading to mutations)

A

C. Antigenic DRIFT since infection with only a single virus strain is required, and the random mutations happen as the virus replicates in the infected person’s cells

How well did you know this?
1
Not at all
2
3
4
5
Perfectly
43
Q

The term Pott’s disease is used to describe which of the following?

A. Disseminated tuberculosis disease
B. CNS tuberculosis
C. Extra-pulmonary tuberculosis
D. Involvement of the spine

A

D. Involvement of the spine

How well did you know this?
1
Not at all
2
3
4
5
Perfectly
44
Q

This is the benefit of CT scan imaging over chest x-ray in visualizing lung abscesses

A. A chest radiograph usually detects a thick-walled cavity with an air-fluid level
B. CT permits better definition and may provide earlier evidence of cavitation
C. Sputum collection for Gram’s stain and culture may yield a pathogen
D. May help distinguish a peripheral lung with pleural effusion from a pleural infection

A

B. CT permits better definition and may provide earlier evidence of cavitation

How well did you know this?
1
Not at all
2
3
4
5
Perfectly
45
Q

Eugene, a 36-year-old software engineer, complained of nasal congestion (with purulent secretions), headache, and sore throat for 2 days. No other symptoms were noted such as fever, malaise, cough, loss appetite, vomiting, loss of taste and smell, and loose stools. He has been working from home since the start of the pandemic. As the number of COVID-19 cases have gone down, their boss has required them to show up at the office at least once a month. This set up has started since July 2021. On further history, he revealed that he was in the office 3 days ago and one of his workmates was noticed to have sneezed a lot on that day. He consulted with you as he wanted to be given antibiotics. Which of the following is an appropriate advice for Eugene?

A. Management for his case would be symptoms based. Antibiotics will be started once there is evidence of bacterial infection, such as the double dip sign.
B. Most URIs are caused by viruses. However, secondary bacterial infection is common therefore, he will be prescribed with an antibiotic for 1 week.
C. Purulent secretions from the nares or throat are indications of bacterial sinusitis or pharyngitis and
therefore, are good indicators of bacterial infection. As he described his nasal secretions as purulent, he should be given antibiotics for 1 week.
D. All of the options are inappropriate

A

no answer on coffee

How well did you know this?
1
Not at all
2
3
4
5
Perfectly
46
Q

Maribel, a 73-year-old retired nurse, is having a 6 months’ tour on a cruise ship with a friend. They starred the tour last February 2020 and is due to end by July 2020. However, the pandemic happened, and they were not allowed to dock in any country. As such, the tour got prolonged. Maribel noted that most of the passengers are from middle- to old-age groups. One day, she noted that some of the patients had complaints of fever, headache, chills, and non productive cough, while others had productive cough. What could be the causative agent in the scenario?

A. Moraxella catarrhalis
B. Streptococcus pneumoniae
C. Legionella pneumophila
D. Haemophilus influenzae

A

C. Legionella pneumophila

How well did you know this?
1
Not at all
2
3
4
5
Perfectly
47
Q

Which of the following is the leading cause of transudative pleural effusions?

A. Left ventricular failure and cirrhosis
B. Pulmonary tuberculosis
C. Nephrosis
D. Metastasis

A

A. Left ventricular failure and cirrhosis

How well did you know this?
1
Not at all
2
3
4
5
Perfectly
48
Q

An accumulation of milky fluid, usually of lymphatic origin, in the pleural cavity

A. Chylothorax
B. Pneumothorax
C. Hemothorax
D. Hydrothorax

A

A. Chylothorax

How well did you know this?
1
Not at all
2
3
4
5
Perfectly
49
Q

Which is more dangerous to human beings, antigenic DRIFT or antigenic SHIFT?

A. Antigenic DRIFT, since this produces the quickest and largest degree of changes in the virus structure and we may not have immunity against it
B. Antigenic SHIFT, the process completely changes the virus, allowing it to jump from one species to another (such as birds into humans). As such, we have no responses in place for the new virus.
C. Antigenic SHIFT, since this produces the quickest and largest degree of changes in the virus structure and we may not have immunity against it
D. Antigenic DRIFT, the small changes make the virus look like something we already have an immune response in place for, but we actually don’t letting the virus hide from the immune responses for a longer period of time

A

B. Antigenic SHIFT, the process completely changes the virus, allowing it to jump from one species to another (such as birds into humans). As such, we have no responses in place for the new virus.

How well did you know this?
1
Not at all
2
3
4
5
Perfectly
50
Q

Which of the following lung carcinoma is highly associated with exposure to tobacco smoke?

A. Squamous cell carcinoma
B. Small cell carcinoma
C. Adenocarcinoma
D. Large cell carcinoma

A

A. Squamous cell carcinoma

How well did you know this?
1
Not at all
2
3
4
5
Perfectly
51
Q

A 40 years old male complains of cough for 6 weeks associated with night sweats and unexplained fever. Which of the following is recommended as first-line diagnostic test for pulmonary TB?

A. Chest x-ray
B. Xpert MTB/RIF assay
C. Smear microscopy
D. Sputum culture

A

B. Xpert MTB/RIF assay

How well did you know this?
1
Not at all
2
3
4
5
Perfectly
52
Q

Which of the following statements regarding PTB treatment and monitoring is correct?

A. Culture and/or smear microscopy is essential in monitoring response to treatment for TB
B. Xpert MTB/RIF should be used to monitor treatment for
C. Regimen 2 (2HRZE/10HR) is indicated for patients with TB meningitis if Xpert result showed MTB not detected
D. To prevent Isoniazid-related neuropathies. Pyridoxine should always be added to every regimen with Isoniazid

A

A. Culture and/or smear microscopy is essential in monitoring response to treatment for TB

How well did you know this?
1
Not at all
2
3
4
5
Perfectly
53
Q

Which of the following is/are considered the primary mechanism/s in the development of emphysema?

A. Autoimmunity
B. Elastin degradation
C. Impaired mucociliary clearance
D. Immune deficiency

A

B. Elastin degradation

How well did you know this?
1
Not at all
2
3
4
5
Perfectly
54
Q

Which of the following refers to the combination of parenchymal lung lesion and nodal involvement?

A. Granuloma
B. Ghon focus
C. Ghon complex
D. Miliary TB

A

C. Ghon complex

How well did you know this?
1
Not at all
2
3
4
5
Perfectly
55
Q

A 65-year-old male presents at the ER with agitation, and body malaise. He is hypertensive with COPD with good compliance to medications. He was noted with hyponatremia and subsequently diagnosed with lung cancer. Which of the following hormones has been responsible to his hyponatremia?

A. Adrenocorticotropic hormone
B. Serotonin
C. Antidiuretic hormone
D. Parathormone

A

C. Antidiuretic hormone

How well did you know this?
1
Not at all
2
3
4
5
Perfectly
56
Q

Which of the following refers to irreversible airway dilatation in a localized area at the lung as a consequence of intrinsic or extrinsic obstruction of the airway?

A. Interstitial lung disease
B. Bronchial asthma
C. Chronic obstructive pulmonary disease
D. Focal bronchiectasis

A

D. Focal bronchiectasis

How well did you know this?
1
Not at all
2
3
4
5
Perfectly
57
Q

You prescribed tiotropium to a COPD patient. You explained that the side effects of the drug are minor and that the most frequently encountered is

A. xerostomia
B. tremors
C. tachycardia
D. palpitations

A

A. xerostomia

How well did you know this?
1
Not at all
2
3
4
5
Perfectly
58
Q

The risk of eventual mortality from COPD is closely associated with reduced levels of

A. PCO2
B. oxygen saturation
C. vital capacity
D. FEV1

A

D. FEV1

How well did you know this?
1
Not at all
2
3
4
5
Perfectly
59
Q

A 33-year-old female was seen at the OPD due to cough. It was described as nonproductive, usually occurring at night or early morning, occasionally triggered by exercise, associated with shortness of breath and dyspnea. Patient is also frequently awakened at night due to aforementioned symptoms. Despite adhering to her Salmeterol + Fluticasone inhaler, symptoms would occur 2-3x per week. Patient also need Salbutamol nebulization 2-3x a week to relieve symptoms. How do you classify the patient’s level of asthma control?

A. Partly controlled
B. Uncontrolled
C. Poorly controlled
D. Controlled

A

B. Uncontrolled

How well did you know this?
1
Not at all
2
3
4
5
Perfectly
60
Q

A diagnosis of Bronchial Asthma can be confirmed through which of the following lung function test results?

A. Reversibility demonstrated by more than 12% and 200 ml increase in FEV1 15 minutes after an inhaled short-acting beta-agonist
B. Reversibility demonstrated by less than 12% and 200 ml increase in FEV1 15 minutes after an inhaled short-acting beta-agonist
C. Reversibility demonstrated by more than 12% and 200 ml increase in FEV1 15 minutes after an inhaled corticosteroid
D. Reversibility demonstrated by more than 12% and 200 ml increase in FEV1 15 minutes after an oral corticosteroid

A

A. Reversibility demonstrated by more than 12% and 200 ml increase in FEV1 15 minutes after an inhaled short-acting beta-agonist

How well did you know this?
1
Not at all
2
3
4
5
Perfectly
61
Q

Which of the following is associated with an increased risk for asthma?

A. High vitamin D diet
B. Obesity
C. High magnesium and selenium in diet
D. Low sodium diet

A

B. Obesity

How well did you know this?
1
Not at all
2
3
4
5
Perfectly
62
Q

Which of the following statements is true?

A. The risk of lung cancer appears to be higher among individuals with low fruit and vegetable intake during adulthood
B. Surgery and radiotherapy are the mainstay therapy for all types of lung cancer
C. Small cell carcinoma tends to remain localized longer than the other forms of lung cancer
D. Small cell carcinoma is the most common form of lung cancer in women and in men

A

A. The risk of lung cancer appears to be higher among individuals with low fruit and vegetable intake during adulthood

How well did you know this?
1
Not at all
2
3
4
5
Perfectly
63
Q

Which of the following is the most common type of asthma and is a classic example of IgE-mediated (type I) hypersensitivity reaction?

A. Drug-Induced Asthma
B. Atopic Asthma
C. Occupational Asthma
D. Non-Atopic Asthma

A

B. Atopic Asthma

How well did you know this?
1
Not at all
2
3
4
5
Perfectly
64
Q

A 27-year-old female had several diagnostic tests as pre-employment management. One of the tests was TST which showed positive. Her chest x-ray was unremarkable as well as her CBC and blood chem. She has which of the following classification?

A. TB disease
B. TB exposure
C. Latent TB infection
D. Presumptive TB

A

C. Latent TB infection

How well did you know this?
1
Not at all
2
3
4
5
Perfectly
65
Q

Which of the following is true regarding smoking in relation to lung cancer?

A. All types of cancer is associated with significant smoking history
B. Long-term former smoker has been shown to have similar risk of developing lung cancer with those who never smoked
C. Smoking cessation after the diagnosis of lung cancer has no noted benefit
D. Second-hand smoke is also an established cause of lung cancer

A

D. Second-hand smoke is also an established cause of lung cancer

How well did you know this?
1
Not at all
2
3
4
5
Perfectly
66
Q

A 65-year-old male came in due to persistent productive cough with thick tenacious sputum for the past year. No fever, chills or weight loss were noted. Patient is a known hypertensive, non asthmatic, non-diabetic, and previously treated for pulmonary tuberculosis for 6 months (2020). Vital signs were within normal limits. Chest and lung exam revealed crackles on both lung fields with occasional wheezing. Sputum gene Xpert results revealed “MTB not detected”. Chest x-ray PA view revealed presence of tram-tracks and atherosclerotic aorta. Which of the following is the likely diagnosis?

A. Community-Acquired Pneumonia, Low Risk
B. Chronic Obstructive Pulmonary Disease
C. Bronchiectasis
D. Pulmonary Tuberculosis, Relapse

A

C. Bronchiectasis

How well did you know this?
1
Not at all
2
3
4
5
Perfectly
67
Q

Which of the following CTD is commonly associated with MVP that is due to fibrillin-1 mutation?

A. Osteogenesis imperfect
B. Fabry’s disease
C. Marfan’s syndrome
D. Ehler-Danlos syndrome

A

C. Marfan’s syndrome

How well did you know this?
1
Not at all
2
3
4
5
Perfectly
68
Q

Which of the following is the pathologic basis for the noted diaphragm paralysis in patients with lung cancer?

A. Extension of tumor into chest wall
B. Phrenic nerve invasion
C. Sympathetic ganglia invasion
D. Tumor spread into pleura

A

B. Phrenic nerve invasion

How well did you know this?
1
Not at all
2
3
4
5
Perfectly
69
Q

In patients with COPD, which of the following is the primary determinant of airflow limitation in the larger airways?

A. Squamous cell metaplasia
B. Goblet cell hyperplasia
C. Mucus gland enlargement
D. Smooth muscle hypertrophy

A

D. Smooth muscle hypertrophy

How well did you know this?
1
Not at all
2
3
4
5
Perfectly
70
Q

A 35-year-old female sough consult at OPD due to cough for almost a month. She was suspected of having pulmonary tuberculosis. Which of the following symptoms supports the diagnosis?

A. Fever almost every afternoon
B. Headache
C. Chest and/or back pains not referable to a musculoskeletal disorder
D. Hematemesis

A

C. Chest and/or back pains not referable to a musculoskeletal disorder

How well did you know this?
1
Not at all
2
3
4
5
Perfectly
71
Q

A 23-year-old male came in due to dyspnea with audible wheezing and productive cough with whitish phlegm for the past 3 days. No fever, chills, chest pain or diaphoresis. Symptoms would usually occur late at night and early morning. Patient is a known asthmatic since childhood and discontinued his inhaler 5 years PTA. Which of the following is the most effective controller for asthma for this patient?

A. Anticholinergics
B. Inhaled corticosteroids
C. Systemic corticosteroids
D. Antileukotrienes

A

B. Inhaled corticosteroids

How well did you know this?
1
Not at all
2
3
4
5
Perfectly
72
Q

Which of the following interventions unequivocally decreases the mortality rate from COPD?

A. Long-acting muscarinic antagonists
B. Oxygen supplementation
C. Long-acting beta-agonists
D. Antibiotics

A

B. Oxygen supplementation

How well did you know this?
1
Not at all
2
3
4
5
Perfectly
73
Q

Which of the following is true of the triggering factors of asthma?

A. Decreased ambient levels of sulfur dioxide, ozone and nitrogen oxides are associated with increased asthma symptoms
B. Exercise is a common trigger of asthma and mechanism is linked to hyperventilation
C. Streptococcus sp. and Staphylococcus sp. are the most common triggers of acute severe exacerbations may invade epithelial cells of the lower as well as upper airways
D. The most common allergen to trigger asthma are Dermatophyte species

A

B. Exercise is a common trigger of asthma and mechanism is linked to hyperventilation

How well did you know this?
1
Not at all
2
3
4
5
Perfectly
74
Q

Which of the following is true of bronchiectasis?

A. Bronchiectasis resulting from infection by non tuberculosis mycobacteria preferentially affects the mid lung fields
B. Predominant involvement of the central airways is commonly due to recurrent immunodeficiency associated infections
C. Allergic bronchopulmonary aspergillosis commonly involves the lower lung fields
D. Bronchiectasis predominantly involves the upper lung fields usually has its source in chronic recurrent aspiration

A

A. Bronchiectasis resulting from infection by non tuberculosis mycobacteria preferentially affects the mid lung fields

How well did you know this?
1
Not at all
2
3
4
5
Perfectly
75
Q

A 24-year-old female, known asthmatic, was seen at the ER due to nonproductive cough for the past 3 days associated with audible wheezing and shortness of breath. She had no coryza, fever, or chills. Pertinent PE revealed: BP of 120/80, HR of 105 bpm, RR of 28-30 cpm, O2 sat of 92-03% at room air, with supraclavicular retractions and wheezing on both lung fields on chest and lung exam. Chest x-ray PA view was unremarkable and COVID-19 RT-PCR test was negative. Which of the following is the most appropriate treatment for this patient?

A. Antileukotrienes
B. Inhaled corticosteroid
C. Theophylline
D. Beta-2 agonist

A

B. Inhaled corticosteroid

How well did you know this?
1
Not at all
2
3
4
5
Perfectly
76
Q

A 68-year-old male who is known to have COPD with post-tuberculosis bronchiectasis, was admitted due to fever and cough (with greenish sputum) and shortness of breathing. Which of the following should be started in this patient?

A. Piperacillin + Tazobactam 4.5gm IV infusion every 8 hours + Azithromycin 500mg once a day
B. Cefuroxime 1.5gm IV every 8 hours + Azithromycin 500mg once a day
C. Ampicillin + Sulbactam 3gm IV every 6 hours + Levofloxacin 500mg once a day
D. Ertapenem 1gm IV infusion once a day + Levofloxacin 500mt once a day

A

B. Cefuroxime 1.5gm IV every 8 hours + Azithromycin 500mg once a day

How well did you know this?
1
Not at all
2
3
4
5
Perfectly
77
Q

According to GOLD 2021, patients with one exacerbation per year with a peripheral blood level ≥ 300 eosinophils/µL identifies patients more likely to respond to

A. LABA
B. LABA and ICS
C. LAMA
D. LABA and LAMA

A

B. LABA and ICS

How well did you know this?
1
Not at all
2
3
4
5
Perfectly
78
Q

Which of the following corticosteroids is currently recommended to be used for COVID-19 patients who require supplemental oxygenation?

A. Dexamethasone
B. Methylprednisolone pulse therapy
C. Hydrocortisone
D. Betamethasone

A

A. Dexamethasone

How well did you know this?
1
Not at all
2
3
4
5
Perfectly
79
Q

Which of the following anti-asthma drugs acts by blocking the cys-LT1-receptors?

A. Montelukast
B. Prednisone
C. Fluticasone
D. Budesonide

A

A. Montelukast

How well did you know this?
1
Not at all
2
3
4
5
Perfectly
80
Q

Which of the following agents is considered as an anti-pneumococcal, non-antipseudomonal beta-lactamase inhibitor?

A. Ceftazidime
B. Ertapenem
C. Piperacillin + Tazobactam
D. Meropenem

A

B. Ertapenem

How well did you know this?
1
Not at all
2
3
4
5
Perfectly
81
Q

The most common mode of transmission of community-acquired pneumonia is via

A. aspiration from the oropharynx
B. droplet spread
C. contiguous extension from an infected pleura
D. hematogenous spread from nearby organs

A

A. aspiration from the oropharynx

How well did you know this?
1
Not at all
2
3
4
5
Perfectly
82
Q

Which of the following medications can be added in a patient with difficult-to-treat asthma who is currently on high dose ICS-LABA?

A. Glycopyrronium
B. Umeclidinium
C. Tiotropium
D. All of the options can be added in this case

A

D. All of the options can be added in this case

How well did you know this?
1
Not at all
2
3
4
5
Perfectly
83
Q

A 68-year-old male who is a previous smoker of approximately 100 pack years sought consult in your clinic due to chronic cough associated with progressive dyspnea especially on exertion. He brought with him his spirometry results (see below). He claims to have been admitted once due to “hubak” in the last month. He was given a series of nebulizations and unrecalled IV medications which improved his symptoms. He was discharged subsequently with instructions to continue nebulization with “Ventolin” at home and to take Prednisone once a day for 5 days. He was instructed to do a spirometry and to bring the result to an internist. At present, he complains that he becomes breathless easily even when he is changing his clothes or taking a bath. Based on the above data, what is the classification of this patient?

A. Gold 3, Class C
B. Gold 2, Class B
C. Gold 2, Class D
D. Gold 3, Class B

A

C. Gold 2, Class D

How well did you know this?
1
Not at all
2
3
4
5
Perfectly
84
Q

Tocilizumab is currently recommended to be used in treatment of COVID-19 in the following patients:

A. COVID-19 patient, 20-year-old male, day 10 of illness, no comorbidities, with O2 sat of 98% at 2LPM via nasalcannula
B. COVID-19 patient who has bronchial asthma controlled, with O2 sat of 96% room air during ambulation
C. COVID-19 severe patient on hemodialysis, crea clearance of 10 ml/min and on high flow nasal cannula at FiO2 of 80%
D. Tocilizumab is recommended for all of these patients

A

C. COVID-19 severe patient on hemodialysis, crea clearance of 10 ml/min and on high flow nasal cannula at FiO2 of 80%

How well did you know this?
1
Not at all
2
3
4
5
Perfectly
85
Q

Which of the following is the most prevalent symptom of a patient with COPD?

A. Recurrent lower respiratory tract infections
B. Progressive and persistent dyspnea
C. Chronic productive cough
D. Chronic sputum production

A

D. Chronic sputum production

How well did you know this?
1
Not at all
2
3
4
5
Perfectly
86
Q

Which of the following Arterial Blood Gas (in room air) values is an indication that a patient with COPD with signs of right-sided heart failure requires supplemental oxygenation at home?

A. pH of less than 7.35
B. PaCO2 more than 45 mmHg
C. PaO2 of 58 mmHg
D. All values must be present to warrant supplemental oxygen at home for this case

A

C. PaO2 of 58 mmHg

How well did you know this?
1
Not at all
2
3
4
5
Perfectly
87
Q

Which of the following may be given to patients with poorly-controlled severe eosinophilic asthma?

A. Azathioprine
B. Mepolizumab
C. Tiotropium
D. Omalizumab

A

B. Mepolizumab

How well did you know this?
1
Not at all
2
3
4
5
Perfectly
88
Q

In the Philippine COVID-19 Living Recommendations, antibody testing is recommended in which of the following settings?

A. To diagnose COVID-19 among suspected patients who had close contact of a known COVID-19 case
B. To determine COVID-19 seroprevalence among patients using IgM assays
C. To determine COVID-19 seroprevalence among adults using lateral flow immunoassays
D. To determine COVID-19 seroprevalence among adults using ECLIA

A

D. To determine COVID-19 seroprevalence among adults using ECLIA

How well did you know this?
1
Not at all
2
3
4
5
Perfectly
89
Q

Which of the following is considered as the most effective controller for the management of asthma?

A. Short-acting beta agonists
B. Inhaled corticosteroids
C. Long-acting beta agonists
D. Long-acting muscarinic agents

A

B. Inhaled corticosteroids

How well did you know this?
1
Not at all
2
3
4
5
Perfectly
90
Q

A 79-year-old male, who is hypertensive and diabetic and with Class D COPD was brought to the emergency room due to difficulty of breathing. He was seen in the emergency room per stretcher, stuporous, in respiratory distress, hypotensive, and with an oxygen saturation of 74% at room air. He was immediately intubated and subsequently mechanically ventilated. He was admitted as a case of CAP-HR. Which of the following empiric antibiotics is best for this patient?

A. Ampicillin-Sulbactam 1.5gm IV every 6 hours + Levofloxacin 750mg tab once a day
B. Ertapenem 1gm IV once a day + Azithromycin 500mg tab once a day
C. Ceftriaxone 2gm IV every 24 hours + Levofloxacin 500mg tab once a day
D. Cefepime 2gms IV infusion every 12 hours + Azithromycin 500mg tab once a day

A

D. Cefepime 2gms IV infusion every 12 hours + Azithromycin 500mg tab once a day

How well did you know this?
1
Not at all
2
3
4
5
Perfectly
91
Q

Which of the following signifies that the specimen is fit for sputum culture and sensitivity?

A. Neutrophils more than 25/hpf; Squamous epithelial cells more than 10/hpf
B. Neutrophils more than 25/lpf; Squamous epithelial cells less than 10/lpf
C. Neutrophils less than 25/hpf; Squamous epithelial cells less than 10/hpf
D. Neutrophils less than 25/lpf; Squamous epithelial cells more than 10/lpf

A

B. Neutrophils more than 25/lpf; Squamous epithelial cells less than 10/lpf

How well did you know this?
1
Not at all
2
3
4
5
Perfectly
92
Q

Which of the following is the antibiotic of choice for MRSA pneumonia?

A. Clindamycin
B. Ampicillin-Sulbactam
C. Linezolid
D. Vancomycin

A

D. Vancomycin

How well did you know this?
1
Not at all
2
3
4
5
Perfectly
93
Q

Which of the following factors strongly supports the combination of inhaled corticosteroids and a LABA in a patient with COPD?

A. History of mycobacterial infection
B. History of childhood asthma
C. Repeated pneumonia events
D. Blood eosinophilia of 300

A

D. Blood eosinophilia of 300

How well did you know this?
1
Not at all
2
3
4
5
Perfectly
94
Q

In spirometry, reversibility to a bronchodilator is defined as

A. an increase in the FVC more than 12% and more than 200 ml from baseline
B. an increase in the FVC more than 12% or more than 200 ml from baseline
C. an increase in the FEV1 more than 12% and more than 200 ml from baseline
D. an increase in the FEV1 more than 12% or more than 200 ml from baseline

A

C. an increase in the FEV1 more than 12% and more than 200 ml from baseline

How well did you know this?
1
Not at all
2
3
4
5
Perfectly
95
Q

Which of the following can be given in patients with uncontrolled severe allergic asthma?

A. Tiotropium
B. Mepolizumab
C. Azathioprine
D. Omalizumab

A

D. Omalizumab

How well did you know this?
1
Not at all
2
3
4
5
Perfectly
96
Q

At which pathologic state of pneumococcal pneumonia will the macrophage reappear as the dominant cell type?

A. Gray hepatization
B. Edema
C. Resolution
D. Red hepatization

A

C. Resolution

How well did you know this?
1
Not at all
2
3
4
5
Perfectly
97
Q

Which of the following is not a component of the Anthonisen’s criteria?

A. Increased sputum production
B. Presence of sputum
C. Increased cough
D. Increased dyspnea

A

C. Increased cough

How well did you know this?
1
Not at all
2
3
4
5
Perfectly
98
Q

A 70yo male, no comorbidities, with known allergy to polysorbate, was in the vaccination area for COVID vaccination. Which of the following is the best vaccine that you will recommended to him?

A. Jansen
B. AstraZeneca
C. Pfizer BioNTech
D. Gamaleya

A

C. Pfizer BioNTech

How well did you know this?
1
Not at all
2
3
4
5
Perfectly
99
Q

Agents belonging to the Carbapenem group are able to inhibit bacterial

A. cell wall synthesis
B. 30s ribosomal subunits
C. 50s ribosomal subunits
D. DNA gyrase activity

A

A. cell wall synthesis

100
Q

Which of the following is the most common risk factor of COPD?

A. Significant smoking history
B. Alpha 1-antitrypsin deficiency
C. Exposure to burning of biomass fuels and/or wood
D. Chronic asthma and airway hypersensitivity

A

A. Significant smoking history

101
Q

Which of the following is the cornerstone in the symptom management of COPD?

A. Inhaled corticosteroids
B. Inhaled antimuscarinic agents
C. Inhaled bronchodilators
D. All of the options are correct

A

C. Inhaled bronchodilators

102
Q

Which of the following is a third-generation cephalosporin with anti-pneumococcal action?

A. Ceftriaxone
B. Ceftazidime
C. Cefixime
D. Cefepime

A

A. Ceftriaxone

103
Q

A 68 yo/F, COPD, was seen at the ER due to cough and fever for 1 day. He is RT-PCR positive with stable V/S. His latest oxygen saturation is 99% in room air. What is her classification based on severity?

A. COVID moderate
B. COVID severe
C. COVID critical
D. COVID mild

A

A. COVID moderate

104
Q

The average decline of FEV1 in non-smoking healthy adults is __ ml/year.

A. 5-10
B. 10-15
C. 15-20
D. 20-25

A

D. 20-25

105
Q

A 69-year-old male who is diagnosed with COPD came in to your clinic for follow-up. He said that he is unable to walk the 50 meters from his front door to his gate due to breathlessness and chest discomfort. Which of the following is the corresponding MMRC grading for this patient?

A. Grade 1
B. Grade 2
C. Grade 3
D. Grade 0

A

Grade 4

106
Q

Which of the following anti-pneumococcal antibiotics acts by inhibiting the DNA gyrase activity in the bacteria?

A. Azithromycin
B. Meropenem
C. Levofloxacin
D. Ceftriaxone

A

C. Levofloxacin

107
Q

Which of the following spirometry results confirm the diagnosis of asthma?

A. Prebronchodilator FEV1/FVC ratio of 0.7
B. Prebronchodilator FEV1 of 0.7
C. Postbronchodilator FEV1 of 0.7
D. Postbronchodilator FEV1/FVC ratio of 0.7

A

D. Postbronchodilator FEV1/FVC ratio of 0.7

108
Q

The following are antivirals currently being used for treatment of COVID-19

A. Leronlimab
B. Remdesivir
C. Tocilizumab
D. Baricitinib

A

B. Remdesivir

109
Q

Which of the following is the preferred medication of choice for a patient with COPD who was admitted 6 months ago for an acute life-threatening exacerbation who presently has MMRC grade of 3 and a CAT score of 20?

A. Formoterol + Budesonide 1 inhalation twice a day
B. Tiotropium 1 capsule through DPI once a day
C. Indacaterol + Glycopyrronium 1 capsule through DPI once a day
D. Vilanterol + Fluticasone 1 inhalation once a day

A

B. Tiotropium 1 capsule through DPI once a day

110
Q

Julianna Hangos, a 40-year-old female sought consult in your clinic due to shortness of breathing for the past 3 days. She claims that she has been having nonproductive cough that worsens at night and throat discomfort for the past 4 weeks, not associated with fever, shortness of breathing, and other symptoms. On the second week of illness, she took Carbocisteine three times a day for 5 days with no note of relief. She began to take Lagundi syrup three times daily 3 days ago, when she noted worsening of the cough with moderate whitish phlegm, and shortness of breathing. She claims that her mother and 3 of her siblings (out of 5) are asthmatic. Which of the following is the diagnostic modality of choice for Ms. Hangos?

A. Daily peak flow measurement for 1 week
B. Spirometry with postbronchodilator study
C. Exhaled nitric oxide (FeNO)
D. Bronchial challenge test using methacholine

A

B. Spirometry with postbronchodilator study

111
Q

Which of the following COVID patients is Baricitinib recommended?

A. 45 yo/M, on chronic hemodialysis with crea clearance at 10 ml/min
B. 69 yo/M with ALT more than 5x elevated
C. 50 yo/F diabetic on day 2 of Remdesivir and currently O2 sat 98% on face mask at 8 LPM
D. Baricitinib is indicated for all these patients

A

C. 50 yo/F diabetic on day 2 of Remdesivir and currently O2 sat 98% on face mask at 8 LPM

112
Q

Which of the following lab parameters are used to guide in using immunotherapy in adult patients with severe COVID 19?

A. Ferritin
B. D-Dimer
C. CBC
D. ESR

A

B. D-Dimer

113
Q

Rapid antigen test (RAT) is recommended in the following situation

A. Symptomatic, on day 10 of illness Should be <7 days
B. Symptomatic and during less than 7 days of onset
C. Asymptomatic, for routine surgical clearance
D. RAT is recommended in all of these cases stated

A

B. Symptomatic and during less than 7 days of onset

114
Q

A 24-year-old female came in for consult due to shortness of breathing. She has asthma since childhood. She uses Salbutamol MDI as needed. She claims to have had on and off cough with shortness of breathing in the past 2 weeks. She has been using her Salbutamol MDI at least 3x a day, with note of temporary relief. However, for the past 3 nights, she noted that she is awakened every 2-3 AM due to chest tightness and shortness of breathing with audible wheezing sounds from her chest. She claims that she would nebulize with Salbutamol + Ipratropium every 30 mins x 2 doses, which afforded relief of symptoms. Which of the following is the most appropriate treatment of this patient?

A. Salmeterol + Fluticasone 50/500mg 1 inhalation 2x a day, Prednisone 30mg/tablet 1 tablet 2x a day for 5-7 days,
B. Salbutamol + Ipratropium 1 nebule 4x a day and Budesonide 1 nebule every 12 hours
C. Salbutamol MDI 2 inhalations every 4 hours, and Prednisone 30mg/tablet, 1 tablet 2x a day for 5-7 days
D. Formoterol + Budesonide 4.5/160mg, 2 inhalations

A

D. Formoterol + Budesonide 4.5/160mg, 2 inhalations

115
Q

55-year-old man is admitted to the hospital with increasing shortness of breath and dry cough for the past few years. He smokes 1.5 packs of cigarettes and drinks about 4 bottles of beer a day. He is constantly “gasping for air” and now walks with difficulty because he becomes breathless after only a few steps. Prolonged expiration with wheezing is noted. PE shows a barrel chest, hyperresonance on percussion, and clubbing of the digits. The patient’s face is puffy and red, and he has pitting edema of the legs. CXR discloses hyperinflation, flattening of the diaphragm, and increased retrosternal airspace. Which of the following best describes the expected histopathology of the lungs in this case?

A. Destruction of the walls of airspaces without fibrosis
B. Prominent bronchial smooth muscle cell hyperplasia
C. Lymphocytes restricted to the interstitium
D. Interstitial fibrosis of the lung parenchyma

A

A. Destruction of the walls of airspaces without fibrosis

116
Q

A 6-year-old male with no prior medical history is brought to the clinic by his mother because his teachers at school noticed he gets tired very quickly during recess. On PE, the patient is sitting comfortably in his chair without any accessory muscle usage. He has mild wheezing throughout all the lung fields. Vital signs were within normal limits. Sputum shows extended granules of eosinophils composed of lysophospholipase enzymes. Patient most likely has

A. emphysema
B. asthma
C. empyema
D. bronchiectasis

A

B. asthma

117
Q

A middle-aged patient with alcohol use disorder is seen in the ER with complaints of fever, chills, night sweats and anorexia. He says he has productive, thick sputum for the past 2 days and has a persistent cough. He describes his sputum as red and viscous. He has a WBC count of 18,000 and a left shift. The rest of the exam is unremarkable. CXR reveals a possible consolidation. What organism is more likely causing the infection?

A. Klebsiella pneumoniae
B. Staphylococcus aureus
C. Mycobacterium tuberculosis
D. Aspergillosis

A

A. Klebsiella pneumoniae

118
Q

An autopsy on a 65-year-old male who died after an acute respiratory tract illness revealed that the lower lobe of his left lung appears grayish brown, firmer than the rest of the lung and has dry cut surfaces. Histological sections show infiltration by macrophages and fibroblasts. How will you label this lesion?

A. Red hepatization
B. Gray hepatization
C. Atelectasis
D. Lung abscess

A

B. Gray hepatization

119
Q

A heavy smoker in the ER is complaining of generalized fatigue. He claims that he is always out of breath and has no more energy. His only medications are bronchodilators, which he rarely uses. Auscultation reveals enhanced resonance of voice sounds to his left chest. What is the most likely cause of this physical finding?

A. Pneumothorax
B. Pneumonia
C. Emphysema
D. Asthma

A

C. Emphysema

120
Q

A 16-year-old male presents with difficulty breathing and severe shortness of breath. Chest auscultation reveals significant bilateral wheeze. BP 130/80, PR 90, Temp 38.4⁰C, RR 30. Pulse oximetry shows an oxygen saturation of 90%. Which of the following statements are true about the airway physiology he is experiencing?

A. Diffusion capacity of the lungs for carbon monoxide (DLCO) is decreased
B. Airways become hyperresponsive
C. FEV1 is increased on spirometry
D. Air trapping occurs in the alveoli

A

B. Airways become hyperresponsive

121
Q

Match the organism with the characteristic respiratory tract infection it is usually associated with Streptococcus pneumoniae:

A. This is more likely to be a nosocomial infection and are more likely to produce a bronchopneumonia with patchy infiltrates
B. Most often seen in immunocompromised patients and is typically bilateral and widespread to the lungs
C. Most often a community-acquired pneumonia (the nursing home counts as a community venue) following a debilitating course like Alzheimer disease
D. Most often produces a granulomatous pattern of inflammation with reticulonodular densities, particularly in the upper lobes

A

C. Most often a community-acquired pneumonia (the nursing home counts as a community venue) following a debilitating course like Alzheimer disease

122
Q

Match the organism with the characteristic respiratory tract infection it is usually associated with Mycobacterium tuberculosis:

A. This is more likely to be a nosocomial infection and are more likely to produce a bronchopneumonia with patchy infiltrates
B. Most often seen in immunocompromised patients and is typically bilateral and widespread to the lungs
C. Most often a community-acquired pneumonia (the nursing home counts as a community venue) following a debilitating course like Alzheimer disease
D. Most often produces a granulomatous pattern of inflammation with reticulonodular densities, particularly in the upper lobes

A

D. Most often produces a granulomatous pattern of inflammation with reticulonodular densities, particularly in the upper lobes

123
Q

Match the organism with the characteristic respiratory tract infection it is usually associated with Pneumocystis jiroveci (carinii):

A. This is more likely to be a nosocomial infection and are more likely to produce a bronchopneumonia with patchy infiltrates
B. Most often seen in immunocompromised patients and is typically bilateral and widespread to the lungs
C. Most often a community-acquired pneumonia (the nursing home counts as a community venue) following a debilitating course like Alzheimer disease
D. Most often produces a granulomatous pattern of inflammation with reticulonodular densities, particularly in the upper lobes

A

B. Most often seen in immunocompromised patients and is typically bilateral and widespread to the lungs

124
Q

Match the organism with the characteristic respiratory tract infection it is usually associated with Staphylococcus aureus

A. This is more likely to be a nosocomial infection and are more likely to produce a bronchopneumonia with patchy infiltrates
B. Most often seen in immunocompromised patients and is typically bilateral and widespread to the lungs
C. Most often a community-acquired pneumonia (the
nursing home counts as a community venue) following a debilitating course like Alzheimer disease
D. Most often produces a granulomatous pattern of
inflammation with reticulonodular densities, particularly in the upper lobes

A

A. This is more likely to be a nosocomial infection and are more likely to produce a bronchopneumonia with patchy infiltrates

125
Q

A 55-year-old man is admitted to the hospital with increasing shortness of breath and dry cough for the past few years. He smokes 1.5 packs of cigarettes and drinks about 4 bottles of beer a day. He is constantly gasping for air and now walks with difficulty because he becomes breathless after only a few steps. Prolonged expiration with wheezing is noted. PE shows a barrel chest hyperresonance on percussion and clubbing of the digits. The patient’s face is puffy and red and he has pitting edema of the legs. CXR discloses hyperinflation, flattening of the diaphragm, and increased retrosternal air space. Which of the following is the appropriate diagnosis?

A. Chronic bronchitis
B. Emphysema
C. Hypersensitivity pneumonitis
D. Asthma

A

B. Emphysema

126
Q

In COPD, eosinophils can be found in

A. lung tissue
B. the blood
C. the sputum
D. all these

A

D. all these

127
Q

A middle-aged alcoholic man is admitted to the hospital in severe respiratory distress. Temp 38.7⁰C, RR 32, BP 130/90. He coughs constantly and expectorates “currant jelly” sputum. CXR shows bilateral diffuse pulmonary consolidation. PE shows bilateral crackles, dullness to percussion over both lung fields, and use of accessory muscles. The patient subsequently dies from complications of bacterial sepsis. What is the appropriate diagnosis?

A. Bronchopneumonia
B. Pulmonary abscess
C. Atypical pneumonia
D. Lobar pneumonia

A

D. Lobar pneumonia

128
Q

A 65-year-old male presents with a cough. The condition started 2 years ago with productive cough lasting for 3 months for 2 consecutive years and is associated with shortness of breath. He is a 20-pack-year smoker. Temp 37.2⁰C, BP 140/90 mmHg, HR 85, RR 25, and O2 sat 93%. PE shows bluish lips and fingernails. Chest and lung exam reveal symmetrical chest expansion, intercostal retractions, and wheezing and crackles on both lung fields. CBC shows a hemoglobin of 14 g/dL, hematocrit 46%, WBC count of 10,000/L, segmenters 50%, lymphocytes 40%, and a platelet count of 200,000/L. CXR shows increased bronchovascular markings and cardiomegaly. Which of the following is the expected pulmonary function test results in patients like him with mucus hypersecretion?

A. Increased FEV1
B. Decreased FEV1
C. Increased FVC
D. Decreased FVC

A

B. Decreased FEV1

129
Q

A high school teacher has had a high fever for a week accompanied by a cough production of yellowish sputum. On PE, T 38.2⁰C, diffuse rales in all lung fields. CXR reveals patchy infiltrates in all lung fields, and there is a 4-cm nodular area of consolidation in the left upper lobe that has an air-filled level. Examination of her sputum reveals numerous neutrophils. Which of the following infectious agents is most likely causing the patient’s pulmonary disease?

A. Staphylococcus aureus
B. Streptococcus pneumoniae
C. Adenovirus
D. Mycoplasma pneumoniae

A

A. Staphylococcus aureus

130
Q

A senior citizen presents at the clinic for dyspnea the past 11 months. The symptoms are pronounced with exertion. No associated orthopnea, chest pain, pedal edema. He is a 30-pack-year smoker. CXR shows hyperinflated lungs with flattened diaphragm. Which of the following factors is associated with poor prognosis of this condition?

A. Female gender
B. Immunocompetent state
C. Bronchial hyperreactivity
D. Obesity

A

C. Bronchial hyperreactivity

131
Q

A previously healthy 5-year-old girl develops a fever along with dyspnea. On PE, temp 37.9⁰C. Her lung fields are clear to auscultation but there are expiratory wheezes. CXR reveals clear lung fields. Lab study shows WBC count 14,480//L. Her dyspnea suddenly worsens, and bronchoscopy reveals bronchi plugged by exudates. These findings are
most consistent with infection by which of the following organisms?

A. Aspergillus flavus
B. Haemophilus influenzae
C. Candida albicans
D. Mycobacterium tuberculosis

A

B. Haemophilus influenzae

132
Q

A teenage female presents to the clinic accompanied by her mother with complaints of occasional wheezing. The wheezing is often accompanied by a cough, which has increased in frequency lately. The mother reports her daughter and son are allergic to dust and her son is being treated for eczema. What is the most likely spirometry reading for this
child?

A. RV decreased, FEV1 increased, FEV1/FVC ratio increased
B. RV increased, FEV1 decreased, FEV1/FVC ratio increased
C. RV increased, FEV1 decreased, FEV1/FVC decreased
D. RV increased. FEV1 decreased, FEV1/FVC decreased

A

C. RV increased, FEV1 decreased, FEV1/FVC decreased

133
Q

A 60-year-old man present to the clinic with chronic productive cough and expectoration of copious sputum. He has history of smoking for the past 30 years. BP 110/80 mmHg, Temp 38.2⁰C, PR 104, RR 30. Lung exam reveals coarse crackles in both lungs. CXR shows increased broncho-vesicular marking on both lungs. Which of the following best describes the pathophysiology of the patient’s condition?

A. There is a permanent and abnormal widening of the bronchi
B. There is permanent enlargement of airspaces
C. It increases forced expiration within the first second
D. Overproduction and hypersecretion of mucus by goblet cells of the respiratory epithelium

A

A. There is a permanent and abnormal widening of the bronchi

134
Q

Which of the following is the most common cause of infections of the upper respiratory tract in children?

A. Coronavirus
B. Influenza virus
C. Rhinoviruses
D. Respiratory syncytial virus

A

C. Rhinoviruses

135
Q

Which of the following solvents is not capable of inactivating SARSCOV-2?

A. Chloroform
B. Peroxyacetic acid
C. 75% ethanol
D. Chlorhexidine

A

D. Chlorhexidine

136
Q

Which of the following associations is correct regarding organ dysfunction in COVID 19?

A. Cardiovascular system: myocarditis
B. Hematologic: disseminated intravascular coagulopathy
C. Central nervous system: encephalitis
D. Hepatobiliary tree: cholestasis

A

A. Cardiovascular system: myocarditis

137
Q

Which of the following conditions present with subacute cough?

A. Post-nasal drip
B. Sinusitis
C. Asthma
D. Pertussis

A

D. Pertussis

138
Q

Which of the following auscultatory findings is OFTEN associated with viral or bacterial bronchitis?

A. Rhonchi
B. Stridor
C. Wheezing
D. Rales

A

A. Rhonchi

139
Q

Cytokine storm often seen in severe forms of COVID-19 associated with the release of high levels of

A. IL 6
B. IL 12
C. IL 1
D. IFN-gamma

A

A. IL 6

140
Q

Which of the following SARSCOV-2 positive individuals will most likely develop severe COVID?

A. 30-year-old male with atopic dermatitis
B. 35-year-old male with obesity and hypertension
C. 28-year-old pregnant G1P0 on her 3rd trimester
D. 20-year-old female with allergic rhinitis

A

B. 35-year-old male with obesity and hypertension

141
Q

Which of the following parameters will classify a patient with COVID 19 as having moderate illness?

A. Oxygen saturation of 95% at fiO2 0.2
B. Oxygen saturation of 95% at fiO2 0.6
C. Lung infiltrates in more than 50% of the lungs
D. Anosmia or dysgeusia

A

A. Oxygen saturation of 95% at fiO2 0.2

142
Q

Upper respiratory infection associated with conjunctivitis may suggest infection with

A. Parainfluenza virus
B. Influenza virus
C. Adenovirus
D. Rhinovirus

A

C. Adenovirus

143
Q

A man is brought into the Emergency Room Department due to stridor. Which of the following is the most likely cause of his condition?

A. Asthma
B. Pneumonia
C. Chronic bronchitis
D. Upper airway obstruction

A

D. Upper airway obstruction

144
Q

Which of the following specimen is appropriate to detect Legionella?

A. Stool
B. Blood
C. Sputum
D. Urine

A

D. Urine

145
Q

The sputum culture from an elderly man confined in the Pulmo Ward yielded growth of Enterobacteriaceae. A review of his history will most likely reveal

A. heart failure
B. chronic use of proton pump inhibitors
C. bronchiectasis
D. seizure disorders

A

A. heart failure

146
Q

An x-ray of the chest revealed pleural effusion in the right basal area. Which of the following PE findings correlates to the presence of this pathology?

A. Bronchial breath sounds on auscultation
B. Decreased vocal fremitus
C. Hyperresonance on percussion
D. Increased tactile fremitus

A

B. Decreased vocal fremitus

147
Q

Hypoxemia often results in patients with pneumonia. This is a direct consequence of

A. bacterial inhibition of vasoconstriction
B. capillary leak
C. alveolar filling
D. increased respiratory drive

A

C. alveolar filling

148
Q

Which of the following characteristics of sputum marks it as an INAPPROPRIATE specimen for culture?

A. More than 25 neutrophils/LPF
B. Absence of blood
C. More than 25 lymphocytes/LPF
D. More than 10 squamous cells/LPF

A

D. More than 10 squamous cells/LPF

149
Q

Which of the following factors is associated with early deterioration in community-acquired pneumonia?

A. Hyperglycemia
B. Tachypnea
C. Thrombocytosis
D. Age less than 40 years old

A

B. Tachypnea

150
Q

Which of the following causes of community-acquired pneumonia can be prevented by administration of vaccine?

A. Moraxella
B. Mycoplasma
C. Staphylococcus aureus
D. Streptococcus pneumonia

A

D. Streptococcus pneumonia

151
Q

History of exposure or travel may give a clue as to the possible etiology of a patient with pneumonia. Which of the following is correctly paired?

A. Stay in a hotel: Moraxella
B. Birds: Francisella
C. Smoking: Staphylococcus
D. Sheep: Coxiella

A

D. Sheep: Coxiella

152
Q

A 30-year-old teacher is suspected to have community-acquired pneumonia. Past history and exposure are all unremarkable. Pertinent findings include tachypnea, temperature of 38.0⁰C, fine rales both lung fields. Chest x-ray revealed infiltrates in the inner lung zones. Which of the following is the most appropriate treatment for this patient?

A. Cefuroxime IV
B. Clindamycin IV
C. Azithromycin PO
D. Amoxicillin-clavulanate PO

A

C. Azithromycin PO

153
Q

Which of the following morphologic changes is highly suggestive of viral etiology of pneumonia?

A. Interstitial mononuclear infiltrates
B. Fibrinosuppurative exudates in the alveoli
C. Neutrophil-rich exudate in the bronchioles
D. Polymorphonuclear infiltrates in the alveolar spaces

A

A. Interstitial mononuclear infiltrates

154
Q

A 3-year-old boy is brought to the emergency room due to sudden onset of difficulty breathing while playing in the neighborhood park. Upon examination you can hear a very audible high-pitched sound every time he inhales. Which among the following is the most likely cause?

A. Tonsillar hypertrophy
B. Foreign body aspiration
C. Bronchial asthma
D. Benign tumor of the proximal airways

A

B. Foreign body aspiration

155
Q

Which of the following is considered as the most effective controller of asthma?

A. Short-acting-beta-2 agonist
B. Cromolyn sodium
C. Omalizumab
D. Inhaled corticosteroid

A

D. Inhaled corticosteroid

156
Q

Which of the following is considered as the characteristic physiologic abnormality in asthma?

A. Airway hyperresponsiveness
B. Fixed airflow obstruction
C. Thickened basement membrane due to subepithelial collagen deposition
D. Airflow limitation

A

A. Airway hyperresponsiveness

157
Q

Which of the following is the most common type of asthma?

A. Occupational
B. Non-atopic
C. Atopic type
D. Drug-induced

A

C. Atopic type

158
Q

Which of the following allergist is most commonly known to trigger asthmatic attacks?

A. House dust mites
B. Cat and dog fur
C. Cockroaches
D. Pollen

A

A. House dust mites

159
Q

Which of the following occurs during the late-phase reaction of IgE mediated asthma?

A. Recruitment of WBCs
B. Vasodilation
C. Bronchoconstriction
D. Increased mucus production

A

A. Recruitment of WBCs

160
Q

Which of the following statements is TRUE?

A. The Clara cells, found in the interalveolar septum, help remove secretions and particulate matter that has penetrated the lungs
B. The club cells are exocrine cells which function to detoxify inhaled foreign substances in the airways
C. Pulmonary surfactant, secreted by type I pneumocytes, lowers the surface tension at the air-epithelium interface
D. Bronchoalveolar fluid, composed of bronchial mucus and alveolar lining fluids, prevents alveolar collapse at exhalation

A

B. The club cells are exocrine cells which function to detoxify inhaled foreign substances in the airways

161
Q

Which of the following cytokines stimulates mucus secretion from bronchial submucosal glands?

A. IL-13
B. IL-10
C. IL-5
D. IL-4

A

A. IL-13

162
Q

In patients presenting with wheezing, the physician should differentiate if it is indeed wheezing or stridor and determine whether the complaint is acute or chronic. Which of the following conditions present with chronic stridor?

A. Foreign body in the trachea
B. Tracheomalacia
C. Aspiration
D. Bronchiectasis

A

B. Tracheomalacia

163
Q

Which of the following inflammatory mediators has a major definitive role in the causation of symptoms of asthma as backed by pharmacologic studies?

A. Acetylcholine
B. Prostaglandin
C. Platelet aggregating factor
D. Histamine

A

A. Acetylcholine

164
Q

Which of the following inflammatory cells is responsible for production of elastase?

A. Lymphocytes
B. CD8+ T cells
C. Eosinophils
D. Neutrophils

A

D. Neutrophils

165
Q

Which of the following derangements is typical of chronic obstructive pulmonary disease?

A. Decreased lung capacity
B. Increased FEV1/FVC ratio
C. Decreased residual volume
D. Reduced forced expiratory flow ratio

A

D. Reduced forced expiratory flow ratio

166
Q

Which of the following agents is proven to decrease mortality rated in patients with COPD?

A. Antibiotics
B. Combination LABA-muscarinic antagonist
C. Inhaled steroids
D. Supplemental oxygen

A

D. Supplemental oxygen

167
Q

Which of the following pathologic types of emphysema is commonly seen in heavy smokers?

A. Centrilobular
B. Panacinar
C. Irregular
D. Paraseptal

A

A. Centrilobular

168
Q

Dyspnea can have pulmonary or extra-pulmonary causes. Which of the following best describe dyspnea in heart failure?

A. Orthopnea
B. Persistent dyspnea
C. Platypnea
D. Nocturnal dyspnea

A

D. Nocturnal dyspnea

169
Q

Which of the following morphologic features in the lung parenchyma is seen in emphysema?

A. Goblet cell hyperplasia
B. Neutrophilic predominant-infiltrates
C. CD8+ cells in the alveolar space
D. Destruction of bronchial walls

A

C. CD8+ cells in the alveolar space

170
Q

Which of the following events occur during the pathogenesis of emphysema?

A. Macrophage mediate impaired repair
B. Destruction of collagen
C. Lymphocytes release proteases
D. Apoptosis of epithelial cells

A

D. Apoptosis of epithelial cells

171
Q

Which of the following manifestations is highly suggestive of poorly compliant lung?

A. Tripod position
B. Exertional dyspnea
C. Cyanosis
D. Tachypnea

A

A. Tripod position

172
Q

Which of the following findings is consistent with dyspnea?

A. Wheezing
B. Hyperresonance on percussion
C. Increased tactile fremitus
D. Egophony

A

B. Hyperresonance on percussion

173
Q

Which of the following signs, when present in a patient with COPD, should warrant an investigation for possible development of lung cancer?

A. Bitemporal wasting
B. Clubbing
C. Cyanosis
D. Loss of subcutaneous adipose tissue

A

B. Clubbing

174
Q

Which extrapulmonary site is most commonly involved in TB?

A. Bone
B. Pericardium
C. Lymph node
D. Meninges

A

C. Lymph node

175
Q

Gerard is a 25-year-old male HIV on antiviral and anti-TB treatment. He visited your clinic for his regular checkup. Which of the following diagnostic test can you test for response to the anti-TB meds started?

A. Smear microscopy
B. GeneXpert
C. Chest x-ray
D. Tuberculin test

A

A. Smear microscopy

176
Q

A 50-kg patient, GG comes to clinic complaining of cough for 2 months. She told Dr. S that she had also been sweating much at night. Chest x-ray was ordered which showed infiltrates on the right apex. What is the correct diagnosis for this patient?

A. Clinically diagnosed EPTB
B. Bacteriologically confirmed PTB
C. Clinically diagnosed PTB
D. Bacteriologically confirmed EPTB

A

C. Clinically diagnosed PTB

177
Q

What is the most widely used regimen used in LTBI chemoprophylaxis?

A. Rifampicin 5mg/kg for 9 months
B. Isoniazid 5mg/kg for 9 months
C. Isoniazid 5mg/kg for 6 months
D. Rifampicin 5mg/kg for 6 months

A

B. Isoniazid 5mg/kg for 9 months

178
Q

Which of the following pulmonary conditions is a leading cause of massive hemoptysis?

A. Bronchiectasis
B. Pneumonia
C. Bronchitis
D. Tuberculosis

A

A. Bronchiectasis

179
Q

A 30-year-old female non-smoker presents with fever and cough for 3 days’ duration. Expectoration of blood streaked sputum prompted this consultation. Physical examination as well as initial CXR is unremarkable. Which of the following is the most appropriate intervention for this case?

A. Treat as bronchitis and follow up after 3 days
B. Suggest chest CT scan since this is more accurate than CXR
C. Do CBC and urinalysis
D. Refer to a specialist for further evaluation

A

A. Treat as bronchitis and follow up after 3 days

180
Q

Recommended by the WHO as the first-line diagnostic test in adults and children with signs and symptoms of active TB

A. Drug susceptibility testing
B. Xpert MTB/Rif assay
C. TB culture
D. Direct sputum smear microscopy

A

B. Xpert MTB/Rif assay

181
Q

Connie, 40 years old, came in due to cough of 3 weeks’ duration with associated night sweats, fever, and weight loss. Apicolordotic x-ray showed infiltrates in both apices, suggestive of tuberculosis. Upon further probing, past medical history revealed intake of anti-TB medications in the past for approximately 25 days. How will you classify this patient as to the history of previous treatment?

A. New case
B. Lost to follow-up
C. Treatment outcome unknown
D. Retreatment case

A

A. New case

182
Q

Which of the following patients is DSSM contraindicated?

A. 92-year-old stroke and bedridden patient
B. 70-year-old male with oral cancer
C. 15-year-old with massive hemoptysis
D. 32-year-old pregnant female with HIV

A

C. 15-year-old with massive hemoptysis

183
Q

The reduced effectiveness of most antibiotics in the treatment of tuberculosis can be attributed to its

A. high permeability of the cell wall
B. high content of mycolic acids
C. formation of spores
D. aerobic characteristic

A

B. high content of mycolic acids

184
Q

The antimalarial drug chloroquine had been controversially reported to treat COVID-19 based on its limited action as a/an

A. anti-inflammatory agent
B. bronchodilator
C. mediator antagonist
D. mucoregulator

A

B. bronchodilator

185
Q

Which of the following conditions necessitates increasing the dose of theophylline in order to meet therapeutic concentration levels?

A. Concomitant use of Rifampicin
B. High protein high carbohydrate level
C. Old age
D. Pneumonia

A

A. Concomitant use of Rifampicin

186
Q

Which of the following are prodrugs that need to be activated by enzymes to become active anti-TB drugs?

A. INF and RIF
B. RIH and PZA
C. INH and PZA
D. ETH and PZA

A

C. INH and PZA

187
Q

Which of these conditions will benefit from a stand-alone LABA?

A. Pneumonia
B. Pulmonary tuberculosis
C. COPD
D. Bronchial asthma

A

C. COPD

188
Q

Which among the following anti-TB medications is the most hepatotoxic?

A. Bedaquiline
B. Isoniazid
C. Rifampicin
D. Pyrazinamide

A

D. Pyrazinamide

189
Q

Mary, a 44-year-old, female, diabetic, with a weight of 40 kg, is on hemodialysis. Her latest creatinine clearance is 20 ml/min. She recently was diagnosed with bacteriologically confirmed PTB. Xpert mtbrif result: MTB detected, rifampicin resistance not detected. Which of the following statements is true with regards to her treatment?

A. ZE is given only daily with same dosing
B. HR is given daily with same dosing
C. HRZE fixed dose combination is given daily
D. All of the options are correct

A

B. HR is given daily with same dosing

190
Q

Which of the following statements is/are true of principles in anti-TB chemotherapy?

A. Long treatment duration permits action on all bacillary populations
B. The use of drug combinations avoid selection of naturally resistant patients
C. Single dose administration to achieve blood peak levels, post antibiotic effect and facilitates supervision
D. All of the options are true

A

D. All of the options are true

191
Q

In patients with asthma, wheezing is characterized by which of the following?

A. Increased expiratory phase
B. Increased inspiratory phase
C. Decreased inspiratory phase
D. Decreased expiratory phase

A

A. Increased expiratory phase

192
Q

A 21-year old medical student, known asthmatic, was brought to the emergency room because of wheezing and shortness of breath. For the past 2 months, she had been having occasional cough and occasional shortness of breath that is relieved by use of her Ventolin inhaler. However, the past 7 days, she complained of runny nose and cough, which then progressed to development of shortness of breath immediately thereafter. She reported her symptoms have occurred 3-4 times in the morning the past 7 days necessitating frequent use of her Ventolin inhaler although she only had 1 episode of asthma exacerbation at night that affected her sleep. She is also becoming short of breath climbing their 3-level dormitory. At the ER, spirometry revealed FEV1 70% and reduced FEV1/FVC. How will you classify this patient as to asthma severity?

A. Severe Persistent
B. Intermittent
C. Mild Persistent
D. Moderate Persistent

A

D. Moderate Persistent

193
Q

A 27/F patient with asthma reports occasional nighttime awakening due to dyspneic episodes but denies any daytime symptoms nor any limitation of activities. She is compliant with her ICS+LABA maintenance medications, which she takes twice daily. How would you classify the patient’s level of asthma control?

A. Uncontrolled
B. Poorly controlled
C. Partly controlled
D. Controlled

A

C. Partly controlled

194
Q

Which of the following suppressive antibiotic protocol is appropriate for cystic fibrosis-associated bronchiectasis?

A. Rotating IV antibiotics
B. Rotating inhalational aminoglycosides
C. Ciprofloxacin once a day on alternate days x 2 weeks
D. Thrice daily amoxicillin x 7 days

A

B. Rotating inhalational aminoglycosides

195
Q

A 68 year-old woman complaints of progressive dyspnea for the past 2 days accompanied with fever and over-all feeling of being ill. She has been having productive cough for the past 2 years. She described the sputum as approximately 1 cup per day, tenacious and yellow to green but without blood and, difficult to expectorate. She reports that she coughs throughout the day which she finds very tiring and embarrassing. Pertinent PE include a febrile state, wheezing and crackles on both lung fields and digital clubbing. Chest Xray show tram-track sign. Which of the following interventions is priority at this point?

A. Anti-inflammatory agents
B. Chest physiotherapy and pulmonary toilette
C. Mucolytics and bronchodilators
D. Antibiotic coverage

A

D. Antibiotic coverage

196
Q

Which of the following is TRUE regarding asthma?

A. Most adult patients with chronic asthma have irreversible airflow obstruction
B. Patients with adult-onset asthma have a high chance of becoming permanently asymptomatic
C. In children, more females are prone to developing asthma than males
D. Signs and symptoms peak at the age of three years

A

D. Signs and symptoms peak at the age of three years

197
Q

Which of the following results of percussion in suggestive of emphysema?

A. Dull
B. Hyperresonance
C. Loud
D. Tympanitic

A

B. Hyperresonance

198
Q

A 4-year-old girl is brought to the emergency room due to sudden onset of difficulty breathing while playing in the neighborhood park. Upon examination you can hear a very audible high-pitched sound every time she inhales. Which among the following is the most likely cause?

A. Benign tumor of the proximal airways
B. Foreign body aspiration
C. Bronchial asthma
D. Tonsillar hypertrophy

A

B. Foreign body aspiration

199
Q

Which of the following is considered as the most effective controller of asthma?

A. Cromolyn sodium
B. Short-acting-beta-2 agonist
C. Inhaled corticosteroid
D. Omalizumab

A

C. Inhaled corticosteroid

200
Q

Which of these statements do not describe the morphology of bronchiectasis?

A. Necrotic changes of the bronchiolar walls often lead to the development of lung abscess
B. Desquamation of lining epithelium and extensive areas of ulceration is most commonly seen in chronic cases
C. Bronchiectatic distal bronchi may appear as cystic structures cross-sectionally
D. Vertically-oriented bronchioles are most severely affected

A

B. Desquamation of lining epithelium and extensive areas of ulceration is most commonly seen in chronic cases

201
Q

Which among the following statements is TRUE regarding atopy as a major risk factor for asthma?

A. The allergens that lead to sensitization are usually proteins that have protease activity
B. Most common allergens related to sensitization are found from cold environments
C. Patients with asthma commonly suffer from atopic dermatitis which may be found in over 80% of asthmatic patients
D. Atopy is due to the genetically determined production of specific IgA antibody, with many patients showing a family history of allergic diseases

A

A. The allergens that lead to sensitization are usually proteins that have protease activity

202
Q

Patients with this type of asthma have negative skin tests and normal serum IgE, later onset of disease, commonly have concomitant nasal polyps, and may be aspirin sensitive:

A. Asthmatic granulomatosis
B. Neutrophilic asthma
C. Intrinsic asthma
D. Eosinophilic asthma

A

C. Intrinsic asthma

203
Q

Jose Hubak came in for his quarterly consultations in your clinic. He claims that he has “asthma attacks” once a week for the past three months. This is usually on a Saturday when he goes running on a trail in Bukidnon. He claims that is usually relieved with 1- 2 inhalations of Formoterol + Budesonide 160/4.5. What is the level of asthma symptom control of Jose Hubak?

A. Partly-controlled
B. Poorly controlled
C. Uncontrolled
D. Well-controlled

A

D. Well-controlled

204
Q

Which of the following tests has the potential for monitoring compliance to inhaled corticosteroid by measuring airway inflammation?

A. FEV1/FVC
B. PC20
C. Fractional exhaled nitric oxide
D. Peak expiratory flow

A

C. Fractional exhaled nitric oxide

205
Q

Which of the following is an appropriate approach to diagnosing patients presenting with wheezing?

A. Work up with spirometry and imaging is indicated for wheezing of acute onset
B. Chronic wheezing should warrant further work up for COPD, heart failure, and bronchial asthma
C. Infection may be due to peritonsillar abscess, pharyngitis, or epiglottitis
D. Foreign body aspiration warrants immediate imaging or endoscopy.

A

B. Chronic wheezing should warrant further work up for COPD, heart failure, and bronchial asthma

206
Q

An Increase in which postbronchodilator parameter denotes a positive reversibility test?

A. FEV1 of 12% or 200ml from baseline
B. FEV1 of 12% and 200ml from baseline
C. FVC of 12% or 200ml from baseline
D. FEV1/FVC of 12% or 200ml from baseline

A

B. FEV1 of 12% and 200ml from baseline

207
Q

Which of the following allergens is most commonly known to trigger asthmatic attacks?

A. Cat and dog fur
B. Cockroaches
C. Dermatophagoides species
D. Pollen

A

C. Dermatophagoides species

208
Q

Which of the following statement is TRUE?

A. The club cells are exocrine cells which function to secrete antimicrobial peptides and cytokines
B. Bronchoalveolar fluid, composed of bronchial mucus and alveolar lining fluids, prevents alveolar collapse at exhalation
C. The Clara cells, found in the interalveolar septum, help remove secretions and particulate matter that has penetrated the lungs
D. Pulmonary surfactant, secreted by Clara Cells, lowers the surface tension at the air-epithelium interface

A

A. The club cells are exocrine cells which function to secrete antimicrobial peptides and cytokines

209
Q

A 50-year old office worker with a 15-pack years cigarette smoking history was brought to the ER because of persistent cough, dyspnea and thick sputum production that often aggravates difficulty of breathing. How should the attending physician at the ER approach this case?

A. Request chest computed tomography and look for tubular structures that are seen close to the pleural space
B. The physician should perform a thorough PE and note for pertinent findings like nail clubbing since this is the most common extrapulmonary sign in bronchiectasis
C. The patient should undergo pulmonary function test which will typically show moderate to severe airflow obstruction
D. Perform serial chest radiography and note for development of new infiltrates and “tram tracks.”

A

A. Request chest computed tomography and look for
tubular structures that are seen close to the pleural space

210
Q

Which among the following statements is TRUE regarding exercise-induced asthma?

A. It typically begins once exercise has started and resolves spontaneously within about 30 minutes
B. The mechanism involved is linked to hyperventilation
C. It may be prevented by prior administration of oral corticosteroids and antileukotrienes
D. It is worse in hot, humid conditions than in cold and dry climates

A

B. The mechanism involved is linked to hyperventilation

211
Q

Which of the following is most common trigger for asthma exacerbation?

A. Change in weather
B. Dust
C. Food
D. Common colds

A

D. Common colds

212
Q

Many inflammatory cells are known to be involved in asthma. Which of the following is recruited as a result of IL5 expression by TH2 cells early in the course of asthma?

A. Neutrophils
B. Macrophages
C. Eosinophils
D. Basophils

A

C. Eosinophils

213
Q

A 60-year-old man presents to the clinic with chronic productive cough and expectoration of copious sputum. He has a history of smoking for the past 30 years. BP 110/80, Temp. 38.2/ C, PR 104, RR 30. Lung exam reveals coarse crackles in both lungs. Chest x-ray shows increase broncho-vascular marking on both lungs. Which of the following best describes the pathophysiology of the patient’s condition?

A. It increases forced expiration within the first second
B. There is overproduction and hypersecretion of mucus by goblet cells of the respiratory epithelium
C. There is permanent enlargement of airspaces
D. There is a permanent and abnormal widening of the bronchi

A

B. There is overproduction and hypersecretion of mucus by goblet cells of the respiratory epithelium

214
Q

A 21-year old medical student, known asthmatic, was brought to the emergency room because of wheezing and shortness of breath. For the past 2 months, she had been having occasional cough and occasional shortness of breath that is relieved by use of her Ventolin inhaler. However, the past 7 days, she complained of runny nose and cough, which then progressed to development of shortness of breath immediately thereafter. She reported her symptoms have occurred 3-4 times in the morning the past 7 days necessitating frequent use of her Ventolin inhaler although she only had 1 episode of asthma exacerbation at night that affected her sleep. She is also becoming short of breath climbing their 3-level dormitory. At the ER, spirometry revealed FEV1 70% and reduced FEV1/FVC. Which of the following is the most appropriate to add to her current medications?

A. High dose inhaled corticosteroids and long acting β2 agonist
B. High dose inhaled corticosteroids, long acting β2 agonist and oral corticosteroids
C. Low dose inhaled corticosteroids
D. Low dose inhaled corticosteroids and long acting β2 agonist

A

D. Low dose inhaled corticosteroids and long acting β2 agonist

215
Q

Which of the following is the most common type of asthma?

A. Atopic
B. Drug-induced
C. Non-atopic
D. Occupational

A

A. Atopic

216
Q

A 34-year old male sex worker sought consult due to cough and occasional dyspnea in a private clinic.
Initial chest Xray showed widespread bronchiectatic changes seen on both lung fields. He was given medications to control his dyspnea and was sent home, however, he was brought to the ER 2 days after for increasing severity of dyspnea. Primary working impression at the ER is HIV/AIDS. Which of these laboratory/ancillary work up shall be done for this specific case on admission?

A. Culture for acid fast bacilli
B. Bronchoscopy
C. Immunoglobulin measurement
D. Sputum gram stain/culture

A

C. Immunoglobulin measurement

217
Q

Which of the following is TRUE about the histology of the lungs?

A. The larger bronchioles are composed of ciliated simple columnar epithelium or simple cuboidal epithelium
B. Type II alveolar cells make up the majority of the alveolar lining and secrete the pulmonary surfactant
C. Bronchiolar exocrine cells contribute to the secretion of surfactant lipoproteins and mucins in the fluid layer on the epithelial surface
D. Clara cells are characterized as non-ciliated columnar cells

A

C. Bronchiolar exocrine cells contribute to the secretion of surfactant lipoproteins and mucins in the fluid layer on the epithelial surface

218
Q

Which of the following LABA has the longest duration of action?

A. Salbutamol SABA
B. Formoterol
C. Salmeterol
D. Vilanterol

A

D. Vilanterol

219
Q

Which of the following cytokines stimulates mucus secretion from bronchial submucosal glands?

A. IL-4
B. IL-10
C. IL-5
D. IL-13

A

D. IL-13

220
Q

Which of the following best characterizes wheezing?

A. It is an inspiratory sound produced in severely narrowed large airways
B. It is caused by severe obstruction of proximal airways
C. It is a musical sound produced primarily in severely narrowed large airways
D. It is a musical sound produced during expiration in narrowed airways

A

D. It is a musical sound produced during expiration in narrowed airways

221
Q

Which of these statements is TRUE of the pathogenesis of bronchiectasis?

A. One mechanism of large airway wall damage in bronchiectasis is the release of reactive oxygen species by the inflamed cells seen in these airways
B. Pseudomonas aeruginosa has been implicated in progressive impairment of mucociliary clearance further damaging the airways
C. Pro-inflammatory cytokines exert its damaging effects on both small and large airways resulting in significant airflow obstruction
D. Whooping cough in patients with cystic fibrosis may lead to substantial airway damage

A

D. Whooping cough in patients with cystic fibrosis may lead to substantial airway damage

222
Q

Which among the following is TRUE about smoking-related ILD?

A. Wheezing is a common PE finding
B. Predominantly occurs in heavy smoker between 50-60 years of age
C. Dyspnea and chest pain are common symptoms
D. Poor prognosis for most cases

A

A. Wheezing is a common PE finding

223
Q

Which among the following treatments of COPD has demonstrated to influence the natural history of patients?

A. Lung volume reduction surgery
B. Lung transplantation
C. Inhaled corticosteroids
D. Pulmonary rehab

A

A. Lung volume reduction surgery

224
Q

Which of the following statements about the clinical manifestations of COPD is correct?

A. Patients with minimal airflow obstruction exhibit use of accessory muscles of respiration.
B. Current evidence demonstrates that most patients have elements of both bronchitis and emphysema and that physical examination does not reliably differentiate the two entities.
C. The most common symptoms in COPD are cough and exertional tachypnea.
D. Activities that allow the patient to brace the arms are particularly difficult for patients with COPD.

A

B. Current evidence demonstrates that most patients have elements of both bronchitis and emphysema and that physical examination does not reliably differentiate the two entities.

225
Q

The main role of __________ is to reduce exacerbations in COPD.

A. Theophylline
B. Oxygen therapy
C. Smoking cessation
D. Inhaled corticosteroids

A

D. Inhaled corticosteroids

226
Q

What type of emphysema is commonly associated with spontaneous pneumothorax in young adults?

A. Panacinar
B. Irregular
C. Centrilobular
D. Paraseptal

A

D. Paraseptal

227
Q

Which of the following findings in a COPD patient is considered to be caused by changes in the large airways, rather than the small airways?

A. Alveolar destruction and perforation
B. Cough and sputum production
C. Atelectasis (lung collapse)
D. Air trapping

A

B. Cough and sputum production

228
Q

Which of the following is the cornerstone in the symptom management of COPD?

A. Inhaled antimuscarinic agents
B. Oral corticosteroids
C. Inhaled corticosteroid
D. Inhaled bronchodilators

A

D. Inhaled bronchodilators

229
Q

Treatment with a systemic steroid is most indicated in which of the following conditions?

A. Asthma
B. Interstitial lung disease
C. COPD
D. Pulmonary tuberculosis

A

B. Interstitial lung disease

230
Q

Which among the following is a high resolution CT image finding of desquamative interstitial pneumonia?

A. Peri-lymphatic nodules
B. Patchy bilateral symmetric ground-glass opacities
C. Prominent interstitial thickening
D. Honeycombing

A

B. Patchy bilateral symmetric ground-glass opacities

231
Q

What is the most common risk factor of COPD?

A. Chronic asthma and airway hypersensitivity
B. Exposure to burning of biomass fuels and/or wood
C. Significant smoking history
D. Alpha 1-antitrypsin deficiency

A

C. Significant smoking history

232
Q

Which of the following refers to dyspnea that is alleviated when lying down and is exacerbated when standing up?

A. Orthopnea
B. Platypnea
C. Paroxysmal nocturnal dyspnea
D. Bradypnea

A

B. Platypnea

233
Q

Which of the following statements regarding the risk factors in COPD is TRUE?

A. Ambient air pollution is a more important risk factor for COPD than cigarette smoking.
B. Pack years is the average number of packs of cigarettes smoked per day divided by the total number of years of smoking.
C. Asthmatics with reduced lung function early in life were more likely to meet spirometric criteria for COPD in early adulthood.
D. Airway responsiveness is the most highly significant predictor of FEV1.

A

C. Asthmatics with reduced lung function early in life were more likely to meet spirometric criteria for COPD in early adulthood.

234
Q

Which of the following is NOT characteristic of COPD?

A. Narrowing of small bronchioles
B. Destruction and enlargement of the lung alveoli
C. Increase in FEV after inhaled albuterol confirms diagnosis
D. Presence of chronic cough and phlegm

A

C. Increase in FEV after inhaled albuterol confirms diagnosis

235
Q

The hallmark of COPD is:

A. Increased lung volumes
B. Hyperinflation
C. Prolonged expiratory phase
D. Airflow obstruction

A

D. Airflow obstruction

236
Q

Which of the following components is responsible for the efferent signals that results in dyspnea?

A. J receptors
B. Motor cortex
C. Chemoreceptors of carotid body
D. Metaboreceptors

A

B. Motor cortex

237
Q

Which of the following effects of cigarette smoke is considered to be responsible for the predisposition to bacterial infection in airways of affected persons?

A. Loss of cilia and impaired macrophage function
B. Presence of metalloproteinases and serine proteinases
C. Elastin degradation and disordered repair
D. Increased B cells and lymphoid follicles present around the airways

A

A. Loss of cilia and impaired macrophage function

238
Q

Which of the following is the initial management for patients with smoking-related ILD?

A. Psychological counseling
B. Early initiation of immunosuppressive agents
C. Oxygen supplementation
D. Smoking cessation

A

D. Smoking cessation

239
Q

Which of the following cardiopulmonary exercise test findings is conclusive of an abnormality of the respiratory system?

A. Increase in dead space or hypoxemia during peak exercise
B. Heart rate is 85% of the of the predicted maximum
C. Ischemic changes in ECG during exercise
D. BP becomes excessively high during exercise

A

A. Increase in dead space or hypoxemia during peak exercise

240
Q

A 58-year-old male, smoker, who works in a factory, complained of chronic cough and shortness of breath. Their company physician was suspecting COPD and requested several laboratory tests. Which of the following findings would most likely support his diagnosis?

A. Paucity of parenchymal markings
B. PCO2 >45 mmHg
C. Decreased lung volumes
D. Reduced FEV1 and FEV1/FVC

A

D. Reduced FEV1 and FEV1/FVC

241
Q

A patient comes in due to shortness of breath upon walking on level ground. Which dyspnea Grade does he belong based on the Modified Medical Research Council Dyspnea Scale?

A. 2
B. 0
C. 3
D. 1

A

D. 1

242
Q

A 69-year-old male who is diagnosed with COPD came into your clinic for follow-up. He said that he is unable to walk the 50 meters from his front door to his gate due to breathlessness and chest discomfort. What is the corresponding mMRC grading for this patient?

A. Grade 2
B. Grade 0
C. Grade 1
D. Grade 3

A

D. Grade 3

243
Q

A 48 yo female seeks consult for dyspnea and cough. She is a known chain smoker who admits that she is able to consume 30 sticks of cigarettes per day. She started smoking when she was 18 yo. How many pack years has she been smoking?

A. 40 years
B. 45 years
C. 35 years
D. 30 years

A

B. 45 years

244
Q

Ruel, 65/M, arrived at ER because of sudden onset of dyspnea which occurred after he went farming. He is a known case of COPD for 5 years. His watcher claimed that he is poorly compliant with his medication. Upon examination, he was awake, tachypneic and follows command. His vital signs were as follows; BP 140/100, HR- 101, RR- 28, Temp- 36.7, o2 sat- 90% at room air. Wheezing on both lung fields were noted upon auscultation. Which among the following treatments may be given to patient’s present condition?

A. Theophylline
B. PDE4 Inhibitors
C. Bronchodilators
D. Inhaled corticosteroids

A

C. Bronchodilators

245
Q

Mr. Reyes, a 68 years-old male, nonsmoker, hypertensive, comes in due to dyspnea at rest for a month now. Pertinent PE findings include: bilateral wheezing, JVP at 10 cm mmHg, bipedal edema. Which is most likely diagnosis?

A. Congestive heart failure
B. Pulmonary embolism
C. COPD
D. Bronchial asthma

A

B. Pulmonary embolism